2020 AMC 10A Problems/Problem 22

Revision as of 22:24, 1 February 2020 by Dragonchomper (talk | contribs) (Solution (Casework))

Problem

For how many positive integers $n \le 1000$ is\[\left\lfloor \dfrac{998}{n} \right\rfloor+\left\lfloor \dfrac{999}{n} \right\rfloor+\left\lfloor \dfrac{1000}{n}\right \rfloor\]not divisible by $3$? (Recall that $\lfloor x \rfloor$ is the greatest integer less than or equal to $x$.)

$\textbf{(A) } 22 \qquad\textbf{(B) } 23 \qquad\textbf{(C) } 24 \qquad\textbf{(D) } 25 \qquad\textbf{(E) } 26$


Solution (Casework)

Expression: \[\left\lfloor \dfrac{998}{n} \right\rfloor+\left\lfloor \dfrac{999}{n} \right\rfloor+\left\lfloor \dfrac{1000}{n}\right \rfloor\]

Solution:

Let $a = \left\lfloor \frac{998}n \right\rfloor$

Notice that for every integer $n \neq 1$,

$\bullet$ if $\frac{998}n$ is an integer, then the three terms in the expression above must be $(a, a, a)$,

$\bullet$ if $\frac{999}n$ is an integer, then the three terms in the expression above must be $(a, a + 1, a + 1)$, and

$\bullet$ if $\frac{1000}n$ is an integer, then the three terms in the expression above must be $(a, a, a + 1)$.

This is due to the fact that $998$, $999$, and $1000$ share no common factors collectively (other than 1).


Note that $n = 1$ doesn't work; to prove this, we just have to substitute $1$ for $n$ in the expression. This gives us $\left\lfloor \dfrac{998}{1} \right\rfloor+\left\lfloor \dfrac{999}{1} \right\rfloor+\left\lfloor \dfrac{1000}{1}\right \rfloor$ = \frac{998}1 + \frac{999}1 + \frac{1000}1 = 2997 = 999 \cdot 3$which is divisible by 3. Therefore, the case$n = 1$does not work.


Now, we test the three cases mentioned above.


<b>Case 1:</b>$ (Error compiling LaTeX. Unknown error_msg)n$divides$998$The first case  does not work, as the three terms in the expression must be$(a, a, a)$, as mentioned above, so the sum becomes$3a$, which is divisible by$3$.


<b>Case 2:</b>$ (Error compiling LaTeX. Unknown error_msg)n$divides$999$Because$n$divides$999$, the number of possibilities for$n$is the same as the number of factors of$999$, excluding$1$.$999$=$3^3 \cdot 37^1$So, the total number of factors of$999$is$4 \cdot 2 = 8$.

However, we have to subtract$ (Error compiling LaTeX. Unknown error_msg)1$, because the case$n = 1$doesn't work, as mentioned previously.$8 - 1 = 7$We now do the same for the third and last case.


<b>Case 3:</b>$ (Error compiling LaTeX. Unknown error_msg)n$divides$1000$Because$n$divides$1000$, the number of possibilities for$n$is the same as the number of factors of$1000$, excluding$1$.$1000$=$5^3 \cdot 2^3$So, the total number of factors of$1000$is$4 \cdot 4 = 16$.

Again, we have to subtract$ (Error compiling LaTeX. Unknown error_msg)1$, for the reason stated in Case 2.$16 - 1 = 15$Now that we have counted all of the cases, we add them.$7 + 15 = 22$, so the answer is$\boxed{\textbf{(A)}22}$.


~dragonchomper

Video Solution

https://youtu.be/Ozp3k2464u4

~IceMatrix

See Also

2020 AMC 10A (ProblemsAnswer KeyResources)
Preceded by
Problem 21
Followed by
Problem 23
1 2 3 4 5 6 7 8 9 10 11 12 13 14 15 16 17 18 19 20 21 22 23 24 25
All AMC 10 Problems and Solutions

The problems on this page are copyrighted by the Mathematical Association of America's American Mathematics Competitions. AMC logo.png